LSAT and Law School Admissions Forum

Get expert LSAT preparation and law school admissions advice from PowerScore Test Preparation.

 Administrator
PowerScore Staff
  • PowerScore Staff
  • Posts: 8917
  • Joined: Feb 02, 2011
|
#32118
Complete Question Explanation

Strengthen (Principle). The correct answer choice is (E)

This scenario asks you to find a rule that, if true, would help to support the columnist's conclusion. The columnist concludes that a certain proposal is unacceptable. To begin, we must know what this proposal is; the proposal is to use extra money from water utility bills to build new roads. Let's make sure we're clear on the whole picture here. Where did the money come from? It came from an additional charge that was initially meant to fund a new dam.

To prephrase, we should try to describe the train of thought here: what does the columnist consider unacceptable and why? The columnist seems to think that using money that was originally earmarked for one purpose for another, unrelated purpose is unacceptable. Since we are looking for information that will support this conclusion, we will need to find a rule that seems to match this reasoning.

Answer choice (A): There is no question of whether customers will know how the money is being used. Nothing clandestine is implied, simply that the use of the funds for roads instead of a dam is unacceptable. This answer choice invites students to make the unwarranted assumption that it's actually saying that customers always deserve to know ahead of time how money will be used, and any other use is unacceptable. It is attractive based on a misreading.

Answer choice (B): We have nothing in the stimulus to suggest that the use for the roads will benefit only some members of the community. Again, the appeal of this answer is based on the introduction of a new, unwarranted assumption, namely that roads ipso facto benefit only some people and not the entire community.

Answer choice (C): This choice introduces yet more outside, unknown considerations that enervate its ability to strengthen the conclusion. You will note that principles or rules generally conform closely to the facts as stated in the stimulus, in other words they make a powerful, direct link between the conditions in the premises as making a certain outcome justified or highly likely. We know nothing of whether utility customers approve or disapprove of roads.

Answer choice (D): Well, since the legislator proposes the new use of the funds, it is probably safe to assume that the different expenditure will have to be approved by the legislature. Otherwise, this answer seems to attempt to get it halfway right, that an additional charge should be imposed only if a certain condition is met. The problem is the approval of the legislature is not the condition we need to meet to make possible an acceptable additional charge.

Answer choice (E): This is the correct answer choice. This choice gives us the right components of our conditional but in contrapositive form, perhaps causing some confusion. Describe what (E) is stating: that any water utility charge must be used on water-related expenditures. New water charge :arrow: used for water expenditures. The contrapositive is used for water expenditures :arrow: new water charge not acceptable. This matches our stimulus, makes our connection, and strengthens the conclusion.
 brcibake
  • Posts: 55
  • Joined: Jul 19, 2017
|
#38583
I decided to choose answer D because E seemed too obvious. When reading the stimulus, I was under the impression that the water utility company had approval to spend the money on only the dam. I thought the it was approved to collect an additional charge and it was approved to spend money on the dam. So using it for something it wasn't approved for is unacceptable. I see what the columnist is saying now but is there any way I could have avoided this misreading?
Thank You
 Adam Tyson
PowerScore Staff
  • PowerScore Staff
  • Posts: 5153
  • Joined: Apr 14, 2011
|
#38616
First things first, brcibake, and that is that you should never reject an answer just because it seems too obvious! Especially in the first 10 questions of an LR section, where the questions tend to be of relatively low difficulty, obvious answer are frequently right answers. Maybe it's obvious because you know what you are doing, and rejecting it is tantamount to second-guessing yourself instead of trusting your skills and know-how?

In later questions, in the mid-teens and low 20s, when I see an answer that seems fairly obvious, I double check it for tricks and traps, as I expect tough questions there, but even then I won't reject an answer solely because it seems too obvious and easy. I have confidence in my abilities, so I trust myself, and you should too.

Now, the problem with D: the stimulus already told us that the water charge was approved, and for what purpose. This answer has nothing to do with how that money will be spent, but only about whether it may or may not be charged to customers. How does telling us we need the legislature's approval to charge the fee strengthen the claim that, in this case, the road proposal is unacceptable? We have the approval already, and answer D does nothing to restrict how we spend the money. Since it has no bearing on how the money is spent, only on whether or not we can collect it in the first place, it's of no use to us.

Again, don't be afraid of obvious answers, especially as early in the section as this one was! Maybe you're just that good!

Keep at it and good luck.

Get the most out of your LSAT Prep Plus subscription.

Analyze and track your performance with our Testing and Analytics Package.